Problemas para calcular la rotación de Wigner para el fotón

Estoy trabajando en el problema 2 del QFT vol. de Weinberg. 1 capítulo 2. El planteamiento del problema es sencillo. Para un observador, un fotón viaja en el y -dirección con polarización lineal en el z -dirección. En la notación de Weinberg, describen el estado como:

Ψ pag = 1 2 [ Ψ pag , + + Ψ pag , ]

dónde pag pag m = ( pag , pag 0 ) = ( 0 , pag , 0 , pag ) , y el ± los subíndices corresponden a ± 1 Estados de fotones de helicidad. Por supuesto, estoy usando la convención de unidades naturales. = C = 1 .

Ahora considere otro observador, viajando a una velocidad v en el z dirección con respecto a la primera. ¿Cómo describe este nuevo observador el mismo estado?

Ya he pasado por las secciones correspondientes en el capítulo que cubre la solución general/formal al problema. Lo que queremos calcular es:

tu ( Λ ) Ψ pag = 1 2 ( Λ pag ) 0 pag 0 [ mi i θ Ψ pag , + + mi i θ Ψ pag , ]

dónde Λ es la transformación de interés de Lorentz: un simple impulso en el z -dirección- y θ = θ ( Λ , pag ) rotación alrededor de la z -eje correspondiente a la rotación general de Wigner.

Así que este problema esencialmente equivale a calcular θ . Esto es sencillo: una rotación de Wigner W ( Λ , pag ) para una partícula sin masa siempre se puede expresar como S ( α , β ) R ( θ ) , dónde R ( θ ) es la rotación alrededor del z -eje que nos interesa, así que simplemente calcule la rotación de Wigner explícita W = L 1 ( Λ pag ) Λ L ( pag ) , y mira el X y bloque, que sabemos que se verá como una rotación.

W = [ porque θ pecado θ . . pecado θ porque θ . . . . . . . . . . ]

Entonces, empiezo con el cálculo. El cuatro impulso estándar que usaré es k m = ( k , k 0 ) = ( 0 , 0 , k , k ) . Usando esto, empiezo por calcular L ( pag ) .

L ( pag ) = R ( k ^ pag ^ ) B ( k | pag | ) = [ 1 0 0 0 0 porque ( π / 2 ) pecado ( π / 2 ) 0 0 pecado ( π / 2 ) porque ( π / 2 ) 0 0 0 0 1 ] [ 1 0 0 0 0 1 0 0 0 0 A B 0 0 B A ] = [ 1 0 0 0 0 0 A B 0 1 0 0 0 0 B A ]

donde he acortado la notación a través de:

A = pag 2 + k 2 2 pag k , B = pag 2 k 2 2 pag k

Ahora calculo la otra parte de la rotación de Wigner, L 1 ( Γ pag ) .

Λ = [ 1 0 0 0 0 1 0 0 0 0 γ v γ 0 0 v γ γ ] , γ 1 1 v 2 Λ pag = ( 0 , pag , pag v γ , pag γ ) L 1 ( Λ pag ) = B ( | Λ pag | k ) R ( Λ pag ^ k ^ ) = [ 1 0 0 0 0 1 0 0 0 0 A B 0 0 B A ] [ 1 0 0 0 0 porque ( ϕ ) pecado ( ϕ ) 0 0 pecado ( ϕ ) porque ( ϕ ) 0 0 0 0 1 ] = [ 1 0 0 0 0 porque ϕ pecado ϕ 0 0 A pecado ϕ A porque ϕ B 0 B pecado ϕ B porque ϕ A ]

donde he acortado la notación a través de:

A = k 2 + γ 2 pag 2 2 γ pag k , B = k 2 γ 2 pag 2 2 γ pag k , broncearse ϕ = 1 v γ

Así que la matriz final de rotación de Wigner es:

W = L 1 ( Λ pag ) Λ L ( pag ) = [ 1 0 . . 0 1 . . . . . . . . . . ]

Entonces θ = 0 , es decir, la polarización no cambia, al final? No tengo otra intuición para este resultado, así que no conozco ningún control de cordura. Parece extraño que Weinberg incluya un problema cuya respuesta final es aburrida.

Respuestas (1)

Ese resultado me parece intuitivo. El significado de la polarización inicial es que no atraviesa un X filtro polarizador. El otro observador estará de acuerdo en que el fotón está bloqueado y también verá el filtro polarizador como un X filtro polarizador, entonces dirán que la polarización es ortogonal a eso. Y eso es exactamente lo que dice su resultado final.